You are on page 1of 5

Combinatorics course week 8

Hoang Nguyen
July 2021

1. Let S = {1, ..., n} and F is a family of subset of S such that no element


is contained in another with m elements. Show that we have
 
n
m≤
b n2 c
Note: Try to solve this problem without using LYM inequality.
2. Let x1 , ..., xn be positive real numbers, with nP> 1. Show that there are
2n
less than √ n
subsets A of {1, ..., n} such that i∈A xi = 1.

3. There is an integer n > 1. There are n2 stations on a slope of a mountain,


all at different altitudes. Each of two cable car companies, A and B,
operates k cable cars; each cable car provides a transfer from one of the
stations to a higher one (with no intermediate stops). The k cable cars
of A have k different starting points and k different finishing points, and
a cable car which starts higher also finishes higher. The same conditions
hold for B. We say that two stations are linked by a company if one can
start from the lower station and reach the higher one by using one or more
cars of that company (no other movements between stations are allowed).
Determine the smallest positive integer k for which one can guarantee that
there are two stations that are linked by both companies.

1 Homework
1. Let n ≥ 2 be an integer. Let S be a subset of {1, 2, . . . , n} such that S
neither contains two elements one of which divides the other, nor contains
two elements which are coprime. What is the maximal possible number
of elements of such a set S?
2. Let an integer n > 1 be given. In the space with orthogonal coordinate
system Oxyz we denote by T the set of all points (x, y, z) with x, y, z are
integers, satisfying the condition: 1 ≤ x, y, z ≤ n. We paint all the points
of T in such a way that: if the point A(x0 , y0 , z0 ) is painted then points
B(x1 , y1 , z1 ) for which x1 ≤ x0 , y1 ≤ y0 and z1 ≤ z0 could not be painted.
Find the maximal number of points that we can paint in such a way the
above mentioned condition is satisfied.

1
3. Let A ⊂ {1, 2, . . . , 4014}, |A| = 2007, such that a does not divide b for
all distinct elements a, b ∈ A. For a set X as above let us denote with
mX the smallest element in X. Find min mA (for all A with the above
properties).
4. Let A be the largest subset of {1, . . . , n} such that for each x ∈ A, x
divides at most one other element in A. Prove that
 
2n 3n
≤ |A| ≤ .
3 4

5. T is a subset of 1, 2, ..., n which has this property : for all distinct i, j ∈ T


, 2j is not divisible by i . Prove that :
4
|T | ≤ n + log2 n + 2
9

6. Let m and n be positive integers and S be a subset with (2m − 1)n + 1


elements of the set {1, 2, 3, . . . , 2m n}. Prove that S contains m+1 distinct
numbers a0 , a1 , . . . , am such that ak−1 | ak for all k = 1, 2, . . . , m.
7. Let X be a set of 100 elements. Find the smallest possible n satisfying the
following condition: Given a sequence of n subsets of X, A1 , A2 , . . . , An ,
there exists 1 ≤ i < j < k ≤ n such that

Ai ⊆ Aj ⊆ Ak or Ai ⊇ Aj ⊇ Ak .

2 Previous exercises solution


2.1 USAMO 2008
Solution (by Evan Chen): We proceed by induction on n. The base case
n = 1 is clear, so suppose n > 1. Let A denote the set of points
 
1
A = (x, y) : x + y + ≥ n − 2 .

2

For any minimal partition P of Sn , let P denote the path passing through
(n − 1, 0). Pick P such that |P ∩ A| is maximal. We claim that in this case
P = A.

Assume not. Starting from (n − 1, 0), walk along P in any direction until
we encounter a vertex v such that v is not adjacent to any vertex in A we have
not already seen. We consider three cases.
1) If v = (0, n − 1) or v = (0, −n), then this is clearly an endpoint of P . In that
case P spans all of A, as desired.
2) If v is an endpoint other than that, then let a be the ”next” vertex in A. By
minimality of P, a must not be an endpoint. Now delete any vertex adjacent

2
to a, and add the edge av. This increases |P ∩ A| without affecting the number
of paths, contradiction.
3) If v is not an endpoint, again let a be the ”next” vertex in A. Delete the
edge joining v to the vertex not in A, and add the edge va. We again arrive at
a contradiction.
This forces A ⊆ P . Clearly this implies P = A. The remainder of Sn is just
Sn−1 , and hence this requires at least n − 1 paths to cover by the inductive
hypothesis. So Sn requires at least n paths.

2.2 USA TST 2009


Consider the tournament result as a directed graph. Note that every complete
directed graph either has a full length cycle or can be divided into two sets A, B
such that every edges between them are directed to B. This can be proved by
considering maximum cycle of the graph.

If we apply this lemma repeatedly, we can divide a complete directed graph


G into some cycles A1 , A2 , · · · , An such that if i < j, every edges among Ai , Aj
are directed to Aj , so it is enough to prove that each cycles have length less than
M + 1. If it can, we can get a desired arrangement of G by aligning points like
A1 , A2 , A3 , · · · and each points in the same cycle by the reverse order of original
cycle. We’ll prove that if there are no n-cycle in a complete directed graph,
any n + 1-cycles also doesn’t exist. if not so, let C be that cycle. Remove one
point {x} in the C. Applying the lemma to C/{x}, we can divide it to A, B,
which satisfies the fact mentioned above... because there are no full cycle. and
by its definition, initial n + 1-cycle should be in the form of x → A → B → x.
Let a be the lastest point of A in the cycle. than the cycle would proceed as
a → b1 → b2 → · · · . However, there is also a → b2 , and this reduces the length
of the cycle by 1. This contradicts the assumption...

3
3 References and hints
3.1 Problem sources
3.1.1 In-class exercises
1. Sperner
2. Pohoatza’s book
3. IMO 2020

3.1.2 Homework
1. Balkan 2005
2. Vietnam TST 2001
3. Vietnam TST 2007
4. Iran TST 2007

5. Iran TST 2009


6. Romania TST 2006
7. China TST 2017

3.2 Ideas
• #poset
• #matching

• #dirichlet
• #elementsandperspectives

4 Theory
This lesson covers the partially ordered sets (posets). Besides the Katona cycle
technique we learned in week 2, we can employ partitioning and extremal ideas
to handle these problems.

An interesting observation is that two finite sets with the same number of ele-
ments are not subsets of each other. Similarly, two sequence with the same sum
would have some restrictions on its intersection. This observation can motivate
some constructions in poset combinatorics.

The most notable idea for this class of problems is the concept of width and

4
height of a family of posets. Dilworth and Mirsky theorem give us a relationship
between these two concepts. Thus, we can employ the switching perspective idea
from week 1.

4.1 Konig and Dilworth


To prove Dilworth’s theorem for a partial order S with n elements, using Kőnig’s
theorem, define a bipartite graph G = (U, V, E) where U = V = S and where
(u, v) is an edge in G when u < v in S. By Kőnig’s theorem, there exists a
matching M in G, and a set of vertices C in G, such that each edge in the
graph contains at least one vertex in C and such that M and C have the same
cardinality m. Let A be the set of elements of S that do not correspond to any
vertex in C; then A has at least n − m elements (possibly more if C contains
vertices corresponding to the same element on both sides of the bipartition) and
no two elements of A are comparable to each other. Let P be a family of chains
formed by including x and y in the same chain whenever there is an edge (x, y)
in M ; then P has n − m chains. Therefore, we have constructed an antichain
and a partition into chains with the same cardinality.

To prove Kőnig’s theorem from Dilworth’s theorem, for a bipartite graph G =


(U, V, E), form a partial order on the vertices of G in which u < v exactly when
u is in U , v is in V , and there exists an edge in E from u to v. By Dilworth’s
theorem, there exists an antichain A and a partition into chains P both of which
have the same size. But the only nontrivial chains in the partial order are pairs
of elements corresponding to the edges in the graph, so the nontrivial chains in
P form a matching in the graph. The complement of A forms a vertex cover in
G with the same cardinality as this matching.

4.2 Theorems
Theorem 1. (Erdos-Szekeres Theorem) If mn + 1 students, all of different
heights, are arranged in a straight line from left to right, prove that there must
either be subsequence of m + 1 students, whose heights from left to right are
increasing; or a subsequence of n + 1 students, whose heights from left to right
are decreasing.
Theorem 2. (Dilworth) If every antichain in a (finite) partially ordered set has
at most m elements, then the set may be partitioned into m chains.
Theorem 3. (Mirsky) If every chain in a partially order set has at most m
elements, then the set may be partitioned into m antichains.

You might also like